自宅で参加できる読書会
NO. 00190937 DATE 2024 05 17

ストロガッツ非線形ダイナミクスとカオス の読書会ページ

ストロガッツ非線形ダイナミクスとカオス(9784621085806)

楽天へのリンク

ストロガッツ非線形ダイナミクスとカオス

著者:スティーヴン・H.ストロガッツ/田中久陽

出版社:丸善出版 (2015年01月30日頃)

ISBN-10:4621085808

ISBN-13:9784621085806

意見、感想、コメントなど

投稿一覧に戻る

P.258の問題番号「7.6.2」への解答

初期値問題$\ddot{x}+x+ \varepsilon x = 0, \ \ x(0)=1, \ \ \dot{x}(0) = 0 $

(a) 初期条件$x(0)=1, \ \ \dot{x}(0) = 0 $の下での$\ddot{x} + (1+\varepsilon) x = 0$の厳密解は$x(t) = \cos t$となる。

(b) 級数展開$x(t,\varepsilon) = x_0(t) + \varepsilon x_1(t) + \varepsilon^2 x_2(t) + \mathcal{O}(\varepsilon^3)$を代入すると、
\[ (\ddot{x}_0 + x_0) + \varepsilon ( \ddot{x}_1 + x_1 + x_0 ) + \varepsilon^2 ( \ddot{x}_2 + x_2 + x_1 )+ \mathcal{O}(\varepsilon^3) = 0 \]となる。この式はすべての十分小さい$\varepsilon$に対して成立するので、$\varepsilon$のそれぞれのべきの項の係数はいずれも$0$になる。したがって、
\[ \begin{align}
\mathcal{O}(1) &: \ddot{x}_0 + x_0 = 0 \\
\mathcal{O}(\varepsilon) &: \ddot{x}_1 + x_1 + x_0 = 0 \\
\mathcal{O}(\varepsilon^2) &: \ddot{x}_2 + x_2 + x_1 = 0
\end{align}
\]となる。同様の理由から、初期条件についても
\[ x_0(0) = 1, \ \ x_1(0) = 0, \ \ x_2(0) = 0, \\
\dot{x}_0(0) = 0, \ \ \dot{x}_1(0) = 0, \ \ \dot{x}_2(0) = 0 \]となる。以下、各オーダーで初期値問題を1つずつ解いていく。
$x_0(0) = 1, \ \ \dot{x}_0(0) = 0$のもとで、$\ddot{x}_0 + x_0 = 0$を解くと、
\[ x_0(t) = \cos t \]が得られる。次に、$x_1(0) = 0, \ \ \dot{x}_1(0) = 0$のもとで、$\ddot{x}_1 + x_1 + x_0 = 0$を解くと、
\[ x_1(t) = -\frac{1}{2} t \sin t \]が得られる。さらに、$x_2(0) = 0, \ \ \dot{x}_2(0) = 0$のもとで、$\ddot{x}_2 + x_2 + x_1 = 0$を解くと、
\[ x_2(t) = \frac{1}{8} t \sin t - \frac{1}{8} t^2 \cos t \]が得られる。

解答者:goodbook 解答日時:2021-02-25 06:07:00

コメントを書き込む

問題解答へのコメント

1

(c) (b)の結果から摂動解は永年項を含んでいる。このようになる理由は、厳密解が十分小さい$\varepsilon$に対して
\[ \begin{align}
x(t) &= \cos \left( t + \frac{1}{2} \varepsilon t - \frac{1}{8} \varepsilon^2 t + \mathcal{O}(\varepsilon^3) \right) \\
&= \cos t \cos \left( \frac{1}{2} \varepsilon t - \frac{1}{8} \varepsilon^2 t + \mathcal{O}(\varepsilon^3) \right) - \sin t \sin \left( \frac{1}{2} \varepsilon t - \frac{1}{8} \varepsilon^2 t + \mathcal{O}(\varepsilon^3) \right)
\end{align}
\]とできるが、ここで、単純に
\[ \begin{align}
\cos \left( \frac{1}{2} \varepsilon t - \frac{1}{8} \varepsilon^2 t + \mathcal{O}(\varepsilon^3) \right) &= 1 - \frac{1}{8} \varepsilon^2 t^2 + \mathcal{O}(\varepsilon^3) \\
\sin \left( \frac{1}{2} \varepsilon t - \frac{1}{8} \varepsilon^2 t + \mathcal{O}(\varepsilon^3) \right) &= \frac{1}{2} \varepsilon t - \frac{1}{8} \varepsilon^2 t + \mathcal{O}(\varepsilon^3)
\end{align}
\]と近似してしまったため。

投稿者:goodbook 投稿日時:2021-02-25 06:08:15

2

(a)の厳密解を書き間違えていました。正しくは
\[ x(t) = \cos \left( \sqrt{1+\varepsilon} t \right) \]です。

投稿者:goodbook 投稿日時:2021-02-26 04:28:55